chike_eze
Thanks Received: 94
Atticus Finch
Atticus Finch
 
Posts: 279
Joined: January 22nd, 2011
 
 
trophy
Most Thanked
 

Q10 - Columnist: A recent research report

by chike_eze Mon Aug 29, 2011 9:45 pm

Go figure. Got this right during full PT, but got it wrong on review.

Vigorous exercise -> reduce risk of certain cardio-resp. illness
therefore,
non-strenuous walking -> reduce risk of certain cardio-resp illness unlikely to be true.

(This almost seems like a mistaken sufficient for necessary flaw. Do I go too far??)

I was down to A and E.

Correct = (E). Author assumes that because new report gives one cause for reducing risk of cardio-resp, then the older report that cites a different cause is unlikely.

(A) -- I think this would have been correct if it had stated "Fails to consider other possibility of reducing risk of cardio-resp. by means other than vigorous exercise"

Kindly provide some feedback to either confirm or deny my reasoning.
 
timmydoeslsat
Thanks Received: 887
Atticus Finch
Atticus Finch
 
Posts: 1136
Joined: June 20th, 2011
 
This post thanked 3 times.
 
trophy
Most Thanked
trophy
First Responder
 

Re: Columnist a recent research report

by timmydoeslsat Mon Aug 29, 2011 11:16 pm

You are correct that answer choice A is not good because it says "other than exercise." Its conclusion is that one should not heed the older studies that shows that nonstrenuous walking can produce the same benefits.

The stimulus sets up a biconditional situation.

Exercising vigorously ---> Significantly lowers chances of developing certain cardiorespiratory illnesses

Significantly lowers chances of developing certain cardiorespiratory illnesses ---> Exercising vigorously

The problem is in how the conclusion is reached. It is that these conditional statements are based on a recent study and we do not know why this study should be taken as gold while the older study should not. We are not told how this study is any better than an older one.
 
farhadshekib
Thanks Received: 45
Elle Woods
Elle Woods
 
Posts: 99
Joined: May 05th, 2011
 
 
trophy
Most Thanked
 

Re: Q10 - A recent research report

by farhadshekib Tue Aug 30, 2011 5:26 pm

timmydoeslsat Wrote:You are correct that answer choice A is not good because it says "other than exercise." Its conclusion is that one should not heed the older studies that shows that nonstrenuous walking can produce the same benefits.

The stimulus sets up a biconditional situation.

Exercising vigorously ---> Significantly lowers chances of developing certain cardiorespiratory illnesses

Significantly lowers chances of developing certain cardiorespiratory illnesses ---> Exercising vigorously

The problem is in how the conclusion is reached. It is that these conditional statements are based on a recent study and we do not know why this study should be taken as gold while the older study should not. We are not told how this study is any better than an older one.


I don't understand why you guys diagrammed the first statement as conditional...

I don't see any conditional language.

The only conditional sentence I see is the second premise (i.e. "only if").
 
timmydoeslsat
Thanks Received: 887
Atticus Finch
Atticus Finch
 
Posts: 1136
Joined: June 20th, 2011
 
 
trophy
Most Thanked
trophy
First Responder
 

Re: Q10 - A recent research report

by timmydoeslsat Tue Aug 30, 2011 5:40 pm

Exercising vigorously significantly lowers chances of developing certain cardiorespiratory illnesses.

What happens if you exercise vigorously?

You significantly lower your chance of developing certain cardiorespiratory illnesses.

Just on that statement alone, the one I have in this post, we do NOT know what happens if you significantly lower your chance of developing such an illness.

However, in the stimulus, it actually informs us that we actually do not what happens if you significantly lower chances, and this will give us a biconditional statement.
User avatar
 
maryadkins
Thanks Received: 641
Atticus Finch
Atticus Finch
 
Posts: 1261
Joined: March 23rd, 2011
 
 
 

Re: Q10 - A recent research report

by maryadkins Wed Aug 31, 2011 4:05 pm

Good discussion.

timmydoeslsat is right--we have a biconditional rule given in the stimulus. He wrote out the first one for you:

timmydoeslsat Wrote:Exercising vigorously significantly lowers chances of developing certain cardiorespiratory illnesses.

What happens if you exercise vigorously?

You significantly lower your chance of developing certain cardiorespiratory illnesses.


We would diagram this as Chike did:

chike_eze Wrote:Vigorous exercise -> reduce risk of certain cardio-resp. illness


But the next sentence tells us that the reduction happens "only if" the exercise is vigorous. We would diagram this:

reduce risk - > vigorous

So you were right to eliminate (A) Chike, but your hypothetical insertion of the word "vigorous" wouldn't make it right. We're told the opposite in the premise--which we take as true.
 
Daniella.owusu
Thanks Received: 5
Vinny Gambini
Vinny Gambini
 
Posts: 12
Joined: December 04th, 2011
 
 
 

Re: Q10 - Columnist: A recent research report

by Daniella.owusu Thu Jun 21, 2012 12:34 pm

I have a question with this problem. I originally chose A, but then I changed it to E becaue A said by means other than exercise. My reasoning for picking E was that the sufficient can be reached without having that particular necessary factor. In other words, something else can come about from exercising vigorously and significantly lowering one's chances of developing cardio- illnesses. I saw that as the sufficient and another necessary for that could have been nonstrenous walking. So the flaw would be not showing why this would no longer suffice over the new study.

Am I wrong to break it down this way? I saw the way other people attacked this problem and my thinking was a bit different. I wanted to make sure it was still an alright reasoning to attack this problem.

Thank you
User avatar
 
maryadkins
Thanks Received: 641
Atticus Finch
Atticus Finch
 
Posts: 1261
Joined: March 23rd, 2011
 
 
 

Re: Q10 - Columnist: A recent research report

by maryadkins Thu Jun 28, 2012 5:15 pm

Daniella.owusu Wrote:My reasoning for picking E was that the sufficient can be reached without having that particular necessary factor. In other words, something else can come about from exercising vigorously and significantly lowering one's chances of developing cardio- illnesses. I saw that as the sufficient and another necessary for that could have been nonstrenous walking. So the flaw would be not showing why this would no longer suffice over the new study.


If something else can come about from exercising vigorously and reducing your risk... great, but that's not what we're talking about. We're talking about how if you do it, you have reduced risk, and if you have reduced risk from exercise, then it's vigorous exercise.

The issue is that the older reports say the opposite. We have no reason to think the new reports are any better than the old. It's actually not a necessary/sufficient conflict, here.

I'd look back over the conversation on this post and see if you can make sense of it. Get back to us if not!
 
shirando21
Thanks Received: 16
Atticus Finch
Atticus Finch
 
Posts: 280
Joined: July 18th, 2012
 
 
 

Re: Q10 - Columnist: A recent research report

by shirando21 Tue Oct 09, 2012 10:17 pm

are we talking about, the part "But exercise has this effect, the report concludes, only if the exercise is vigorous" is not inferred based on the research report, thus, the "thus...."part is also not right?

Why can't D be correct? is it because it does not address to the issue/core/reasoning of the argument?
 
s.atrmachin3
Thanks Received: 4
Vinny Gambini
Vinny Gambini
 
Posts: 17
Joined: March 05th, 2013
 
 
 

Re: Q10 - Columnist: A recent research report

by s.atrmachin3 Wed Sep 11, 2013 11:06 pm

Bump.

I see why (E) is correct, but I do not see how (A) is incorrect without our making a large assumption.

From the discussion above, it seems like Timmy and Mary are suggesting that (A) is incorrect because of the bi-conditional presented in the stimulus. But isn't it important to note the qualification offered for the second half of the bi-conditional ("but exercise has this effect...")?

In the above posts, "but exercise has this effect... only if the exercise is vigorous" is diagrammed as:

lowers chances of CR illness ---> exercise is vigorous

But that doesn't seem to be an accurate representation of what is said. In my opinion, a more accurate diagram would be:

exercise + lowers chances of CR illness ---> exercise is vigorous

The reason I think this distinction is important (let's see if I can state this clearly) is because this premise is used to justify the conclusion that we shouldn't heed studies that show that nonstrenuous walking yields the same benefits. Which begs the question: Is nonstrenuous walking exercise? The argument assumes that it is, and that strikes me as a very real flaw, on par in degree with that in answer choice (E).

Unless I am misunderstanding the reasoning above for eliminating answer choice (A), it seems that we are making a clear jump in logic by assuming that nonstrenuous walking is exercise.

If someone could offer further insight into and discussion of this problem and my analysis, I'd greatly appreciate it.

Lance
User avatar
 
maryadkins
Thanks Received: 641
Atticus Finch
Atticus Finch
 
Posts: 1261
Joined: March 23rd, 2011
 
 
 

Re: Q10 - Columnist: A recent research report

by maryadkins Sun Sep 15, 2013 3:43 pm

Ha! Smart! Yes, if non-strenuous walking were not exercise, then (A) would correctly identify a flaw with this question. So yep, we have to call it exercise. (I mean, I'd argue that it is exercise, so it's not "assuming"...)
User avatar
 
ttunden
Thanks Received: 0
Atticus Finch
Atticus Finch
 
Posts: 146
Joined: August 09th, 2012
 
 
 

Re: Q10 - Columnist: A recent research report

by ttunden Fri Aug 29, 2014 5:13 pm

Yea I will agree with the other poster here

I got this wrong because I didn't classify walking as exercising. The older study just said that by walking you can achieve a lower chance of developing the cardio illness. I can see how it was E. During the test, I had down A and E as my two contenders.

So is there anything else wrong with A? because to me, during the test, I would not consider walking an exercise. I think a lot of other people would agree
User avatar
 
tommywallach
Thanks Received: 468
Atticus Finch
Atticus Finch
 
Posts: 1041
Joined: August 11th, 2009
 
 
 

Re: Q10 - Columnist: A recent research report

by tommywallach Wed Sep 03, 2014 9:11 pm

Ha! I beg to differ. Of course walking is exercise. I don't know anyone who would disagree with that!

-t
Tommy Wallach
Manhattan LSAT Instructor
twallach@manhattanprep.com
Image
 
PaigeG29
Thanks Received: 0
Vinny Gambini
Vinny Gambini
 
Posts: 2
Joined: July 11th, 2017
Location: University of California Berkeley
 
 
 

Re: Q10 - Columnist: A recent research report

by PaigeG29 Sun Sep 16, 2018 2:58 pm

ttunden Wrote:Yea I will agree with the other poster here

I got this wrong because I didn't classify walking as exercising. The older study just said that by walking you can achieve a lower chance of developing the cardio illness. I can see how it was E. During the test, I had down A and E as my two contenders.

So is there anything else wrong with A? because to me, during the test, I would not consider walking an exercise. I think a lot of other people would agree



exercise is defined as:
a : regular or repeated use of a faculty or bodily organ
b : bodily exertion for the sake of developing and maintaining physical fitness trying to get more exercise

But further, even if we did suppose that walking was not exercise, that wouldn't fully eliminate A.
(A) fails to consider the possibility that the risk of
developing certain cardio-respiratory illnesses
can be reduced by means other than exercise

If walking is not exercise, then this argument indeed does consider the possibility that risk is reduced by something other than exercise (walking), so it does not FAIL TO CONSIDER something else, as it mentions the study and specifically says that it should not be believed (but as I don't think that walking can be disputed as exercise, I think this is a digression from the real issues of the problem).

And that's how we get to the real flaw of the argument: it gives us no reason why we should believe the recent research cited in this argument, rather than "older studies" (plural, so that would seem more reason to question why we would throw out potentially numerous studies which show walking to reduce heart disease), and are expected to just throw these out because of vague explanations of new research proving otherwise. Okay, so we have two mutually exclusive claims: that vigorous exercise is the ONLY exercise that reduces risk, and older evidence which shows that walking ALSO reduces risk (and so would mean that vigorous exercise is NOT the ONLY exercise that reduces risk. In order to judge between these claims, we would need at least some type of explanation as to why the other studies were mistaken, or why the new research is more reliable, etc. -- but this arguments provides no such explanations or justifications. It just tells us "not heed older
studies purporting to show that nonstrenuous walking yields the same benefits."

Thus, the correct answer is
E) fails to show that a certain conclusion of the
recent report is better justified than an
opposing conclusion reached in older studies
 
HeatherF897
Thanks Received: 0
Vinny Gambini
Vinny Gambini
 
Posts: 7
Joined: April 25th, 2019
 
 
 

Re: Q10 - Columnist: A recent research report

by HeatherF897 Tue May 21, 2019 10:35 pm

Different question -

If the justification for this question is "we don't know the new study is better than the old study so we can't conclude that the old study shouldn't be followed" - how is this not a premise detractor, aka something we're generally not supposed to do?
User avatar
 
ohthatpatrick
Thanks Received: 3808
Atticus Finch
Atticus Finch
 
Posts: 4661
Joined: April 01st, 2011
 
 
 

Re: Q10 - Columnist: A recent research report

by ohthatpatrick Thu May 23, 2019 1:11 pm

A few things on that point:

1. You did a great job of hedging the advice -- "It's something we're generally not supposed to do."

Cool, but that still leaves room for exceptions, when we DO "go against" a premise. Most of our thinking about LSAT is couched in terms of TENDENCIES / PRESUMPTIONS IN FAVOR OF _____ , but not rigid absolutes. We have to always think flexibly, deal with the question at hand, and find the best available answer. If that means going against a usual tendency, so be it. It's pretty impossible to get above 170 on this test if we don't use flexible thinking a handful of times on each test.


2. "Detracting from the premise" vs. "Falsifying a premise"

The advice you've (hopefully) heard is that LSAT isn't wanting us to ever contradict a premise and say, "Nuh-uh, that's not true." But detracting from premises is actually the name of the game on most Weaken / Flaw questions. We're saying, "That's true, but it's not compelling or relevant because ____ ".

Think about it in the courtroom context. One lawyer might introduce eyewitness testimony "Ms. Jackson says she saw Sammy at the scene of the crime" or introduce a survey "80% of the residents of surveyed opposed getting rid of the dog park".

The other lawyer can de-value these pieces of evidence without falsifying them:
- "Ms. Jackson has terrible eyesight"
(this doesn't deny that Ms. Jackson claims to have seen Sammy)

- "The survey only asked residents who were currently at the dog park"
(this doesn't deny that the survey found 80% of respondents opposed getting rid of the dog park)

De-valuing these pieces of evidence Weakens the argument / weakens the first lawyer's case.


3. The advice you've gotten about "Don't go against a premise" is not something LSAT has ever said is a rule.

We, who have taken millions of LSATs, simply observe that LSAT does not (usually) go against a premise. It's almost unheard of, so we often overdramatize it as "a rule". But there's no rule. There are a handful of questions in which the correct answer straight up questions the validity of a premise. So since it's only 5 out of every 1000 assumption family problems (if that), we espouse it as "a rule", but stay flexible.